- Thu Jul 13, 2017 2:13 pm
#37234
Question #8 is a Local question with the condition added that O is earlier than M. As a starting point this doesn't tell us much, as either M placement, 3 or 5, would still be possible and thus multiple outcomes allowed. That's also why this makes sense as a Could Be True EXCEPT question: a number of things could still be true with O ---- M.
Specifically we're asked to determine an option that CANNOT go in 5 (that's the "could except" part). To attack it, consider both options, with M 3 and M 5, thinking about that fifth position.
Clearly if M is in 5 then that's the end of the story for that option: M is in the fifth spot, so we're done. And that tells us answer choice (B) is a possibility and thus incorrect.
Now think of the other scenario where O is ahead of M: M in 3. That tells us our first four pieces! With M in 3 we have P in 1, and to get O earlier than M we have to put O into 2. Finally, we know O is two away from L, so with O in 2 we'd have L in 4:
P O M L (N , S , T) [NST here are uncertain, but fill the final three spaces, 5-7]
So either M is 5th, or one of N, S, and T is 5th. That rules out answers (C), (D), and (E), leaving us with answer choice (A) as correct: L can never go fifth.
Another way to approach this would be to test the answer themselves. Imagine what would happen with L in 5. First, that means M is in 3. Has to be if someone else take 5. And then O would either go 7, and thus not be ahead of M (so that's no good here), or O would go 3...but M is already there! So L in 5 leaves no way for O to be ahead of M, and again is the right answer to this question.
Specifically we're asked to determine an option that CANNOT go in 5 (that's the "could except" part). To attack it, consider both options, with M 3 and M 5, thinking about that fifth position.
Clearly if M is in 5 then that's the end of the story for that option: M is in the fifth spot, so we're done. And that tells us answer choice (B) is a possibility and thus incorrect.
Now think of the other scenario where O is ahead of M: M in 3. That tells us our first four pieces! With M in 3 we have P in 1, and to get O earlier than M we have to put O into 2. Finally, we know O is two away from L, so with O in 2 we'd have L in 4:
P O M L (N , S , T) [NST here are uncertain, but fill the final three spaces, 5-7]
So either M is 5th, or one of N, S, and T is 5th. That rules out answers (C), (D), and (E), leaving us with answer choice (A) as correct: L can never go fifth.
Another way to approach this would be to test the answer themselves. Imagine what would happen with L in 5. First, that means M is in 3. Has to be if someone else take 5. And then O would either go 7, and thus not be ahead of M (so that's no good here), or O would go 3...but M is already there! So L in 5 leaves no way for O to be ahead of M, and again is the right answer to this question.
Jon Denning
PowerScore Test Preparation
Follow me on Twitter at https://twitter.com/jonmdenning
My LSAT Articles: http://blog.powerscore.com/lsat/author/jon-denning
PowerScore Test Preparation
Follow me on Twitter at https://twitter.com/jonmdenning
My LSAT Articles: http://blog.powerscore.com/lsat/author/jon-denning